Determine which of the four levels of measurement​ (nominal, ordinal,​ interval, ratio) is most appropriate for the data below. Types of restaurants (fast food, organic food, sea food, etc.)A. The ratio level of measurement is most appropriate because the data can be ordered, differences (obtained by subtraction) can be found and are meaningful, and there is a natural starting point. B. The nominal level of measurement is most appropriate because the data cannot be ordered. C. The interval level of measurement is most appropriate because the data can be ordered, differences (obtained by subtraction) can be found and are meaningful, and there is no natural starting point. D. The ordinal level of measurement is most appropriate because the data can be ordered, but differences (obtained by subtraction) cannot be found or are meaningless.

Answers

Answer 1

Answer:

B. The nominal level of measurement is most appropriate because data cannot be ordered.

Step-by-step explanation:

Nominal scale is used when there is no specific order scale and data can be arranged according to name. Ordinal scale requires variables to be arranged in specific order. For fast food restaurant the best scale used is nominal scale as variables can be arranged according to their name without specific order.


Related Questions

Intravenous fluid bags are filled by an automated filling machine. Assume that the fill volumes of the bags are independent, normal random variables with a standard deviation of 0.08 fluid ounces.
(a)What is the standard deviation of the average fill volume of 22 bags?
(b)The mean fill volume of the machine is 6.16 ounces, what is the probability that the average fill volume of 22 bags is below 5.95 ounces?
(c)What should the mean fill volume equal in order that the probability that the average of 22 bags is below 6.1 ounces is 0.001?

Answers

Answer:

a) 0.0171 fluid ounces.

b) 0% probability that the average fill volume of 22 bags is below 5.95 ounces

c) The mean should be of 6.153 fluid ounces.

Step-by-step explanation:

To solve this question, we need to understand the normal probability distribution and the central limit theorem.

Normal Probability Distribution:

Problems of normal distributions can be solved using the z-score formula.

In a set with mean [tex]\mu[/tex] and standard deviation [tex]\sigma[/tex], the z-score of a measure X is given by:

[tex]Z = \frac{X - \mu}{\sigma}[/tex]

The Z-score measures how many standard deviations the measure is from the mean. After finding the Z-score, we look at the z-score table and find the p-value associated with this z-score. This p-value is the probability that the value of the measure is smaller than X, that is, the percentile of X. Subtracting 1 by the p-value, we get the probability that the value of the measure is greater than X.

Central Limit Theorem

The Central Limit Theorem establishes that, for a normally distributed random variable X, with mean [tex]\mu[/tex] and standard deviation [tex]\sigma[/tex], the sampling distribution of the sample means with size n can be approximated to a normal distribution with mean [tex]\mu[/tex] and standard deviation [tex]s = \frac{\sigma}{\sqrt{n}}[/tex].

For a skewed variable, the Central Limit Theorem can also be applied, as long as n is at least 30.

Standard deviation of 0.08 fluid ounces.

This means that [tex]\sigma = 0.08[/tex]

(a)What is the standard deviation of the average fill volume of 22 bags?

This is s when n = 22. So

[tex]s = \frac{\sigma}{\sqrt{n}}[/tex]

[tex]s = \frac{0.08}{\sqrt{22}}[/tex]

[tex]s = 0.0171[/tex]

(b)The mean fill volume of the machine is 6.16 ounces, what is the probability that the average fill volume of 22 bags is below 5.95 ounces?

We have that [tex]\mu = 6.16[/tex]. The probability is the p-value of Z when X = 5.95. So

[tex]Z = \frac{X - \mu}{\sigma}[/tex]

By the Central Limit Theorem

[tex]Z = \frac{X - \mu}{s}[/tex]

[tex]Z = \frac{5.95 - 6.16}{0.0171}[/tex]

[tex]Z = -12.3[/tex]

[tex]Z = -12.3[/tex] has a p-value of 0.

0% probability that the average fill volume of 22 bags is below 5.95 ounces.

(c)What should the mean fill volume equal in order that the probability that the average of 22 bags is below 6.1 ounces is 0.001?

[tex]X = 6.1[/tex] should mean that Z has a p-value of 0.001, so Z = -3.09. Thus

[tex]Z = \frac{X - \mu}{s}[/tex]

[tex]-3.09 = \frac{6.1 - \mu}{0.0171}[/tex]

[tex]6.1 - \mu = -3.09*0.0171[/tex]

[tex]\mu = 6.153[/tex]

The mean should be of 6.153 fluid ounces.

Calls to a customer service center last on average 2.8 minutes with a standard deviation of 1.4 minutes. An operator in the call center is required to answer 75 calls each day. Assume the call times are independent. What is the expected total amount of time in minutes the operator will spend on the calls each day

Answers

Answer:

The expected total amount of time the operator will spend on the calls each day is of 210 minutes.

Step-by-step explanation:

Normal Probability Distribution:

Problems of normal distributions can be solved using the z-score formula.

In a set with mean [tex]\mu[/tex] and standard deviation [tex]\sigma[/tex], the z-score of a measure X is given by:

[tex]Z = \frac{X - \mu}{\sigma}[/tex]

The Z-score measures how many standard deviations the measure is from the mean. After finding the Z-score, we look at the z-score table and find the p-value associated with this z-score. This p-value is the probability that the value of the measure is smaller than X, that is, the percentile of X. Subtracting 1 by the p-value, we get the probability that the value of the measure is greater than X.

n-values of normal variable:

Suppose we have n values from a normally distributed variable. The mean of the sum of all the instances is [tex]M = n\mu[/tex] and the standard deviation is [tex]s = \sigma\sqrt{n}[/tex]

Calls to a customer service center last on average 2.8 minutes.

This means that [tex]\mu = 2.8[/tex]

75 calls each day.

This means that [tex]n = 75[/tex]

What is the expected total amount of time in minutes the operator will spend on the calls each day

This is M, so:

[tex]M = n\mu = 75*2.8 = 210[/tex]

The expected total amount of time the operator will spend on the calls each day is of 210 minutes.

Weekly demand for a certain brand of a golf ball at The Golf Outlet is normally distributed with a mean of 35 and a standard deviation of 5. The profit per box is $5.00. Write an Excel formula that simulates the weekly profit:
= 5 * 35 * NORMSINV(RAND())
= 5* NORMINV(RAND(), 35, 5)
= 5 * RANDBETWEEN(5, 35)
= NORMINV(RAND(), 5 * 35, 5)

Answers

Answer:

= 5 * NORMINV(RAND(), 35, 5)

Step-by-step explanation:

From the given information:

The total weekly profit is achieved by the multiplication of the unit profit (5) and the weekly demand.

Here, the weekly demands obey a normal distribution where the mean = 35 and the standard deviation = 5.

Using the Excel Formula:

The weekly profit can be computed as:

= 5 * NORMINV(RAND(), 35, 5)

A jacket costs $154.85. There is a 45% discount. What is the new price of the jacket.

A.) $68.68
B.) $85.17
C.) $224.53

Answers

b is is the best answer :) good luck

Answer:

B) $85,167

Step-by-step explanation:

u got discount 45% so u just have to pay 55% of it

cost = 55% x $154,85 = $85,1675

Stan knows that segment AB∥segment CD. He wants to use the definition of a parallelogram to prove that quadrilateral ABCD is a parallelogram. Which equation can he use?

Answers

Answer:

[tex]\frac{q - r}{m- n} = \frac{p - s}{m - n}[/tex]

Step-by-step explanation:

Given

See attachment for parallelogram

Required

Proof that ABCD is a parallelogram

We know that opposite sides are equal and parallel.

First, we calculate the slope of BC

[tex]m = \frac{y_2 - y_1}{x_2 - x_1}[/tex]

So, we have:

[tex]m = \frac{q - r}{m- n}[/tex]

Next, the slope of AD using:

[tex]m = \frac{y_2 - y_1}{x_2 - x_1}[/tex]

So, we have:

[tex]m = \frac{p - s}{m - n}[/tex]

For ABCD to be a parallelogram; then:

[tex]\frac{q - r}{m- n} = \frac{p - s}{m - n}[/tex]

Use Pythagorean Theorem to find each missing length
please help with the steps

Answers

The first answer is going to be 15.42 (so b) and the second is going to be 10.67 (so a)

Answer:

25 is A and 26 is B

Step-by-step explanation:

25) a²+b²=c²

missing side can be=b

to find the missing side subtract 6.7² from 12.6²

b²=12.6²-6.7²

b²=158.76-44.89

the square root of b²= the square root of 113.87

b=10.67

the missing side is equal to 10.7(1d.p)

26) a²+b²=c²

c= hypotenuse

10.8²+11²=c²

116.64+121=c²

c²=237.64

the square root of c²= the square root of 237.64

c=15.42(2d.p)

the hypotenuse is=15.4

The places that I have "the square root of" you must replace it with the square root sign. I'm using my phone so I wasn't sure how to insert a square root sign.

What is the inverse function of y = 2x - 8

Answers

Answer:

Step-by-step explanation:

y = 2x-8

2x = y+8

x = 0.5y+4

inverse function: y = 0.5x+4

You are having a birthday party and are inviting 6 friends. You have 9 cupcakes, and you are going to share the cupcakes fairly among you and your 6 friends.
Which equation describes how many cupcakes each of you will receive?

Answers

Answer:

split the other three in half

Step-by-step explanation:

Ivan runs a cake shop. Renting the
shop costs him $1600 per month,
and he makes a profit of $16 on each
cake he sells. Ivan wants a profit of at
least $2000 a month.

Answers

He’s gotta sell atleast 125 cakes
He’s gotta sell 125 cakes

Q23. Find the value of k, if x = 2, y = 1 is a solution of the equation 2x + 3y = k.​

Answers

Answer:

k=7

Step-by-step explanation:

2x+3y=k

2(2)+3(1)=k

4+3=k

k=7

Answer:

7.

Step-by-step explanation:

Substitute x = 2 and y = 1 into the given equation:

2(2) + 3(1) = k

4 + 3 = k

k = 7.

The product of three consecutive numbers is divisible by ​

Answers

Answer:

6

Step-by-step explanation:

The product of three consecutive numbers is divisible by ​6

Let us say the numbers are x, x+1 , x+2

if x = 1,

Product of the three consecutive numbers,

(1)(2)(3)

=> 6, which is divisible by 6

if x = 2,

Product of the three consecutive numbers,

(2)(3)(4)

=> 24, which is divisible by 6

Similarly if we take any 3 consecutive numbers their product will be divisible by 6.

Is triangle XYZ = ABC ? If so, name the postulate that applies. A. Congruent - ASA B. Congruent - SAS C. Might not be congruent D. Congruent - SSS

Answers

The answer is the SAS postulate. The two triangles have two pairs of corresponding, congruent sides, and the included angles are congruent. And thus, triangles XYZ and ABC are congruent by the SAS postulate.

HELP PLEASE MATH PROBLEM

Answers

Answer:

x=41

Step-by-step explanation:

LM =JM

154=4x-10

154+10=4x

164=4x

164/4=4x/4

41=x

hope this is helpful

the third option , x=41 !!

Find the percent of decrease from 46 songs to 41 songs. Round to the nearest tenth of a percent if necessary.
percent of decrease
%

Answers

Answer:

10.9 %

Step-by-step explanation:

46 - 41 = 5

5/46 * 100% = 10.8695652174%

Rounded

10.9 %

6 Write 89.4945 correct to (a) nearest whole number, [1] (b) two decimal places. ​

Answers

Answer:

a)89

b)89.45

Step-by-step explanation:

Y=x^3+x what's the domaine and range

Answers

domain is  (- infinity, infinity)

range is (- infinity, infinity)

which elements in the following set are integers -8,3/4,-0.18,0,0.16,5,-2/7,6

Answers

Answer:

345

Step-by-step explanation:

Can anyone help me please ????

Answers

Hey there! The topic for this problem is Limit of Function!

As for the question, we are given the quadratic function and we have to find the limit, the value that approaches to a.

[tex] \large \boxed{lim_{x \longrightarrow a} f(x)}[/tex]

We call this, "The limit of f(x) when x approaches a."

Then you may ask, "How do we find the limit of function?". That is a very nice question! The answer to your problem is just substitute x-value in. Although this substitution method only applies when the approaching value doesn't make the denominator to 0. I believe that in the beginning of Limit topic, we learn how to find or evaluate the basic limit that only requires substitution.

So from the question, we receive:

[tex] \large{lim_{x \longrightarrow 2} ( {x}^{2} - 3x - 1)}[/tex]

Next step is to substitute x = 2 in the function.

[tex] \large{lim_{x \longrightarrow 2} ( {2}^{2} - 3(2) - 1)}[/tex]

Evaluate the value.

[tex] \large{lim_{x \longrightarrow 2} ( 4 - 6 - 1)} \\ \large{lim_{x \longrightarrow 2} ( - 3)}[/tex]

Cancel the limit out and there you have it!

[tex] \large \boxed{ - 3}[/tex]

Answer

The limit of quadratic function when x approaches 2 is -3.

Now whenever you learn limit, you must know that limit is when we substitute the approaching value. That means x —> 2 is not x = 2 but x approaches 2.

Regarding the limit, any questions and doubts can be asked through comment and I will get back to you soon!

Thank you for using Brainly and I hope you have a fantastic day! Good luck on the assignment.

Find the Diameter of the circle, whose radius is 17 cm.

Answers

Answer:

34 cm

Step-by-step explanation:

The radius is half of the diameter, so 17 cm is half of 34 cm.

Diameter = 34 cm

Three bags contain 3 red, 7 black; 8 red, 2 black, and 4 red & 6 black balls
respectively. 1 of the bags is selected at random and a ball is drawn from it. If the ball
drawn is red, find the probability that it is drawn from the third bag.

Answers

Answer:

[tex]Probability = \frac{4}{15}[/tex]

Step-by-step explanation:

B1 = first bag

B2= second bag

B3 = third bag

Let A = ball drawn is red

Since, there are three bags.

Probability of choosing one bag=  P(B1) = P(B2) = P(B3) = 1/3.

From B1: Total balls = 10

3 red + 7 black balls.

Probability of drawing 1 red ball from it , P(A) = 3/10.

From B2: Total balls = 10

8 red + 2 black

Probability of drawing 1 red ball is, P(A) = 8/10

From B3 : Total Balls = 10

4 red + 6 black

Probability of drawing 1 red ball, P(A) = 4/10 .

To find Probability given that the ball drawn is red, that the ball is drawn from the third bag by Bayes' rule.

That is , P(B3|A)

                     [tex]=\frac{\frac{1}{3} \times \frac{4}{10}} { \frac{1}{3} \times \frac{3}{10} + \frac{1}{3} \times\frac{8}{10} + \frac{1}{3} \times \frac{4}{10}}[/tex]

   

                    [tex]=\frac{4}{30} \times \frac{30}{15}\\\\=\frac{4}{15}[/tex]

Therefore, the probability that it is drawn from the third bag is 4/15.

Answer:

4/15

Step-by-step explanation:

Solution of conditional probability problem:

Given:

Bags (3R,7B), (8R,2B), (4R,6B)

Let

P(R,i) = probability of drawing a red AND from bag i

P(R, 1) = 3/10 * (1/3) = 3/30

P(R, 2) = 8/10 * (1/3) = 8/30

P(R, 3) = 4/10 * (1/3) = 4/30

Let

Let P(R) = probability of drawing a red from any bag

P(R) = sum P(R,i)  for i = 1 to 3   using the addition rule

= 3/30 + 8/30 + 4/30

= 15/30

= 1 / 2

Conditional Probability of drawing from the third bag GIVEN that it is a red

= P(3 | R)

= P(R, 3) / P(R)

= 4/30 / (1/2)

= 8/30

= 4 / 15

(Since all bags contain 10 balls, by intuition, 4 red from third / 15 total red = 4/15)


Malachy rolls a fair dice 720 times.
How many times would Malachy expect to roll a five?​

Answers

Answer:

120 times

Step-by-step explanation:

On a dice, there are 6 sides.

Since one of these sides is a 5, the chance of rolling a five is 1/6.

Find how many times Malachy can expect to roll a five by multiplying 720 by 1/6:

720(1/6)

= 120

So, Malachy can expect to roll a five 120 times

What translation maps ABC to A'B'C'?

Answers

The answer is D because if u trans it up 4 then C and B won’t be in the right place. So and rotation 90 ccw will do the job.

A positive real number is 5 more than another. When - 10 times the smaller is added to the square of the larger, the result is 57. Find the numbers.

Answers

Answer:

4√2 and 5+4√2

Step-by-step explanation:

Let the two numbers be x ad y

Smaller = y

Bigger = x

If a positive real number is 5 more than another, then;

x = 5 + y ... 1

When - 10 times the smaller is added to the square of the larger, the result is 57, then;

-10y + x² = 57 ...2

Substitute 1 into 2;

-10y + (5+y)² = 57

-10y + 25+10y+y² = 57  

y²+25 = 57

y² = 57 - 25

y² = 32

y = √32

y = 4√2

Since x = 5 + y

x = 5 + 4√2

Hence rhe numbers are 4√2 and 5+4√2

Find the equation of the straight line that passes through the points (1, 10) and (3, 2)
ANSWER ASAP

Answers

Answer:

y = -4x+14

Step-by-step explanation:

First find the slope

m = (y2-y1)/(x2-x1)

m = (2-10)/(3-1)

   =-8/2

   = -4

The slope intercept form of a line is

y = mx+b where m is the slope and b is the y intercept

y = -4x+b

Substitute a point into the equation

10 = -4(1)+b

Add 4 to each side

14 = b

y = -4x+14

How tall is the average human baby ?

Answers

Full term- 20” (50cm)
Normal range 18” to 22”

How tall is the table?
120cm
90cm
I

Answers

The table is 120 cm tall

The values of variables, such as the height of the table can be found by writing equations of their relationships

The height of the table is 105 cm

The reason the above height value is correct is as follows;

Known parameters:

The diagram shows a table, a cat and a mice

Let x, represent the height of the table, let y represent the height of the cat, and let z represent the height of the mice

From the given diagram, we have;

Height of the table + Height of the cat - Height of the mice  = 120 cm

x + y - z = 120...(1)

Height of the table + Height of the mice - Height of the cat   = 90 cm

x + z - y = 90...(2)

Adding equation (1) to equation (2) gives;

x + y - z + (x + z - y) = 120 + 90 = 210

x + y - z + (x + z - y) = 210

However;

x + y - z + (x + z - y) = x + x + y - y - z + z = 2·x

∴ x + y - z + (x + z - y) = 2·x = 210

x = 210/2 = 105

Therefore;

The height of the table, x = 105 cm

Learn more about word problems leading on simultaneous equations here:

https://brainly.com/question/16513646

PLEASE HELP!! graph the circle whose equation is (x-6)^2 + (y+2)^2 =4

Answers

Answer:

Y= -x^2+12x-36

Step-by-step explanation:

HELP HELPPP!!!у- 3
|
у+
у- 3
3
What is the common denominator of y+
3
in the complex fraction
5 2
9* Зу
?
Зу(у – 3)
у(у – 3)
Зу
О 3

Answers

Answer:

The common denominator of [tex]y + \frac{y-3}{3}[/tex] is 3

Step-by-step explanation:

Given

The complex fraction

Required

The common denominator

To solve this, we need not consider the whole complex fraction.

We only consider

[tex]y + \frac{y-3}{3}[/tex]

Take LCM

[tex]y + \frac{y-3}{3} = \frac{3y - (y-3)}{3}[/tex]

Single out the denominator, i.e. 3

Hence, the common denominator of [tex]y + \frac{y-3}{3}[/tex] is 3

Somebody please help me asap​

Answers

Answer:

sum of angles in a triangle = 180°

180-(90+21)

= 69

pls am I correct

Your answer is going to be 69

c+12<16
what will be the answer

Answers

Answer:

[tex]c < 4[/tex]

Step-by-step explanation:

Move the constant to the right-hand side and change its signs:

[tex]c < 16 - 12[/tex]

Subtract the numbers:

[tex]c < 16 - 12 = c < 4[/tex]

Other Questions
How many atoms are in 490 moles of barium? The volume of air in the lungs is controlled by the A school bus is transporting athletes home from an athletics meeting. Of these athletes, 15% took part in hurdles, 20% took part in the long jump event, 35% took part in the relay races, 30% ran the 100m sprint and 25% took part in the high jump event.Explain how this is possible. A petrol can is a rectangular prism with base measurements 15 cm by 30 cm. If the can has capacity 18 liters, find its height. A person takes part in a medical trial that tests the effect of a medicine on a disease. Half the people are given medicine and the other half are given a sugar pill, which has no effect on the disease. The medicine has a 60% chance of curing someone. But, people who do not get the medicine still have a 10% chance of getting well. There are 85 people in the trial and they all have the disease. Talwar takes part in the trial, but we do not know whether he got the medicine or the sugar pill. Draw a tree diagram of all the possible cases. What is the probability that Talwar gets cured? cortHere are ten numbers:3 7 2 4 7 5 7 1 8 8Write down the mode.b)Work out the median.c)Calculate the mean.What is the range? Pls answer both Pls pls Suppose that the total value of dividends to be paid by companies in the Narnian stock market index is $100 billion. Investors expect dividends to grow over the long term by 5% annually, and they require a 10% return. Now a collapse in the economy leads investors to revise their growth estimate down to 4%. By how much should market values change What is lassaigne's test? The basic question between Lincoln and Congress at the end of the CivilWar was...4 points O is the centre of the circle, EF is a tangent, angle BCE = 28, angle ACD = 31Line AB = line ACWrite down the size ofa) angle BACb) angle ABCc) angle ADCd) angle COB thanh iu ca ting vit l g? Please help if you know how to solve itThank you! An organelle that is not found in this illustration of a cell would beA. the cell wallB. chromosomeC. mitochondriaD. the cell membrane The Texas Consolidated Electronics Company is contemplating a research and development program encompassing eight research projects. The company is constrained from embarking on all projects by the number of available management scientists (40) and the budget available for R&D projects (S300,000). Further, if project 2 is selected, project 5 must also be selected (but not vice versa). Following are the resources requirement and the estimated profit for each project.Project Expense Management Estimated Profit ($1,000s) Scientists required (1,000,000s) 1 50 6 0.30 2 105 8 0.85 3 56 9 0.20 4 45 3 0.15 5 90 7 0.50 6 80 5 0.45 7 78 8 0.55 8 60 5 0.40 Formulate the integer programming model for this problem and solve it using the computer. FIRST ANSWER I WILL GIVE BRAINLEST The sound from a trumpet radiates uniformly in all directions in 20C air. At a distance of 5.00 m from the trumpet the sound intensity level is 52.0 dB. The frequency is 587 Hz. (a) What is the pressure amplitude at this distance explain how a lever can act as a force multiplier Jason and Donny painted a house and received $1.200. To complete the painting job Jason painted 4 hours 25 minutes and Donny spent 2 hours and 15 minutes. If they split the $1.200 in proportion to the amount of time each spent painting, how much did Donny receive? How many solutions exist for the system of equations in the graph?